Author Topic: Woodward's effect  (Read 803073 times)

Offline birchoff

  • Full Member
  • **
  • Posts: 273
  • United States
  • Liked: 125
  • Likes Given: 95
Re: Woodward's effect
« Reply #760 on: 09/25/2016 07:56 pm »
I have a couple questions about this solution. I have not gotten around to studying general relativity in detail, so I can't directly critique the calculations, but there are a few implications this brings up.

Are these calculations based on general relativity as it is generally known, or are there some tweaks/ additional assumptions?

As I understand it, the explanation for energy/momentum balance is remote interaction with the other objects in the universe. There are 2 ways I can understand this:
1. Instantaneous interaction with remote objects - this would cause problems with causality.
2. Gravitational waves, which as I understand them have the same Energy/momentum constraints as photon thrusters.

I could see the energy/momentum ratio for gravity waves being modified if you don't take the weak-field limit, but that brings up how any non-planet sized device could be outside the weak field limit.

On a related note, considering the size of the constant G, even if the device was only as efficient for propulsion as a photon thruster, being able control and put significant energy energy into gravitational waves could open up all sorts of interesting applications.

Does the theory really predict meaningful magnitude of the forces, or is it just far enough to say that there is a force in a given direction? From what you said I believe it is the former, but I want to check since I know this could be a very hard calculation to do.

the energy-momentum balance is due to instantaneous interaction with the mass of the rest of the universe. What Hoyle and Narlikar theory provides is a Theory of Gravitation that leverages the ideas in Wheeler Feynman Absorber Theory as a mechanism for explaining how the mass in the outer regions of the universe have the ability to communicate with the mass here. Woodward was able to do mostly the same thing with GR because from his background as a historian he believes (and provides documented proof) that Einstein intended for GR to be machian. It just never had a way to explain the instantaneous communication. Which is why the latest theory papers out of their Lab are talking more about Hoyle Narliker theory of gravitation. As of the papers that came out last year. They have rederived the mach effect equations for at least the first term (the space drive). I would need to re read to see if the term, dubbed the worm hole term. Is still there.

In the end, if correct the instantaneous interaction is carried out by a mechanism that allows all mass to emit advanced/retarded waves through time.

Offline Rodal

  • Senior Member
  • *****
  • Posts: 5911
  • USA
  • Liked: 6124
  • Likes Given: 5564
Re: Woodward's effect
« Reply #761 on: 09/25/2016 09:19 pm »
...As of the papers that came out last year. They have rederived the mach effect equations for at least the first term (the space drive). I would need to re read to see if the term, dubbed the worm hole term. Is still there....
Woodward's derivation of  an equation for mach's principle has two components: a part that is similar to the EM Drive but different in operating detail; and a second part that leads to navigable wormholes.

While currently mucking about with wormholes experimentally is beyond the capabilities of anyone on earth- Woodward  (rightly, I think) says that if the first half of his equation is supported by experiment then the wormhole bit must be correct as well....
In my derivation I show that the splitting of the expression into both of those terms is a) mathematically unnecessary and b) of no mathematical benefit.  I found a mathematical expression that rigorously takes into account both terms and enables a solution of the equations under rigorous, and explicit conditions.   What you call the worm hole term involves the square of the first derivative with respect to time while what you call the impulse term involves the second derivative with respect to time.  To have one without the other would need to have a linear time variation such that the first derivative with respect to time is constant and such that the second derivative with respect to time is zero.  Such a time variation is artificial.  The time variation used in Woodward's experiments has been a harmonic, for example a Cosine variation of the voltage V=Vo Cos[omega t], where obviously both derivatives are present: - Vo omega Sin[omega t] and Vo omega^2 Cos[omega t].  In general, there is no mathematical benefit to split them apart and consider them separate.  Furthermore, in the experiments they have been conducting, both terms are present  ;)
« Last Edit: 09/25/2016 11:08 pm by Rodal »

Offline Rodal

  • Senior Member
  • *****
  • Posts: 5911
  • USA
  • Liked: 6124
  • Likes Given: 5564
Re: Woodward's effect
« Reply #762 on: 09/25/2016 11:14 pm »
...Are these calculations based on general relativity as it is generally known, or are there some tweaks/ additional assumptions?...
Your question is not precise enough to understand what you mean by "generally known". My derivation is based on Hoyle-Narlikar's theory of gravitation. Dr. Lance Williams, at the Estes Park Workshop presented another derivation based on a linearization of Einstein's General Relativity.  Prof. Martin Tajmar derived these terms from a linearization of Einstein's General Relativity using yet another approach.

I don't know what you mean by "additional tweaks".  Nothing additional is needed to be added to General Relativity.  The only reason why many people are unfamiliar with these terms is because most treatments disregard, "ab initio", the derivatives with respect to time in general relativity.  The possible objection is whether such time rate terms can be confirmed by experiments.  Whether they are present in Nature or whether they are just a mathematical solution, a mathematical curiosity.

There are many unusual solutions in general relativity (not surprisingly  :) ), for example when the black hole solution first appeared as a mathematical solution, Einstein's first reaction was that it must be a mathematical non-physical solution.  Only much later the black hole solutions were confirmed to be present in Nature, contrary to Einstein's intuition.  Ditto for other solutions.  On the other hand, Goedel's rotating universe and its strange implications for the flow of time still looks like a mathematical solution that may not be present in the Universe we live in. (*)

Naturally, to proceed to derive a force, once you obtain the expressions that are dependent on the time rates, you need to derive the equation of motion.  But an equation of motion is not a "tweak".  Equations of motion follow from a Lagrangian (for no damping).  I also take into account damping. 

...
As I understand it, the explanation for energy/momentum balance is remote interaction with the other objects in the universe. There are 2 ways I can understand this:
1. Instantaneous interaction with remote objects - this would cause problems with causality.
2. Gravitational waves, which as I understand them have the same Energy/momentum constraints as photon thrusters.

....
Please read Hoyle-Narlikar's papers.  Advanced-retarded waves an application of Wheeler–Feynman absorber theory to general gravitation.

We are not talking here about sending information with advanced-retarded waves or superluminally so there are no issues of causality as you infer above.  Such issues of causality and the arrow of time can only occur when attempting to send information or macro processes.  In my derivation I explicitly state that the velocity of the mass particles is explicitly assumed to be much smaller than the speed of light.  Rather, the issue for Feynman's theory was self-interaction.

Again, if you prefer, rather than dealing with the nonlinear, fully covariant formulation of Hoyle-Narlikar, you can linearize general relativity and obtain such time-dependent terms.  While these solutions are possible solutions from relativity, the ultimate question is whether they are physically possible in Nature.  At the Estes Park Conference, there were three groups in 2 continents that had independently confirmed non-null results with experimental measurements of such a force, in the same direction as predicted by theory.  The experiments also agreed as to similar magnitudes (for the specific material and geometry of the stack being tested) .More experiments, are obviously needed.

To my mind, one of the best experiments conducted up to date was the experiment conducted by Fearn and Woodward a couple of years ago, with equal tail and head masses that showed no significant force.  This is in excellent agreement with the solution I obtained that shows that there is no force for a completely symmetric system.

-------------

(*) For curiosity, I just googled Goedel rotating universe to see whether any interesting new papers have been writen about it and found out this paper where somebody tried to look at it as an explanation for the Pioneer anomaly  :)  https://arxiv.org/ftp/arxiv/papers/0908/0908.4067.pdf

It looks like everybody is throwing the kitchen sink to try to explain by means other than just a thermal radiation propulsion issue  :)
« Last Edit: 09/26/2016 12:04 am by Rodal »

Offline meberbs

  • Senior Member
  • *****
  • Posts: 3096
  • Liked: 3379
  • Likes Given: 777
Re: Woodward's effect
« Reply #763 on: 09/26/2016 12:00 am »
Thank you for this information. My first question wasn't stated rigorously because I do not know enough about the details of general relativity to ask it properly, but you provided the information I was interested in.

I think I would want to pick up a general relativity textbook and get more comfortable with the basics before I jump into the papers you mentioned, but once I find the time to do so, I will come back to this and study the implications in more detail. I have the math background where I could probably read them now, but I wouldn't understand what it means physically. (at least to my satisfaction, since I can be picky, I want to understand things inside and out)

Offline Rodal

  • Senior Member
  • *****
  • Posts: 5911
  • USA
  • Liked: 6124
  • Likes Given: 5564
Re: Woodward's effect
« Reply #764 on: 09/26/2016 12:07 am »
Thank you for this information. My first question wasn't stated rigorously because I do not know enough about the details of general relativity to ask it properly, but you provided the information I was interested in.

I think I would want to pick up a general relativity textbook and get more comfortable with the basics before I jump into the papers you mentioned, but once I find the time to do so, I will come back to this and study the implications in more detail. I have the math background where I could probably read them now, but I wouldn't understand what it means physically. (at least to my satisfaction, since I can be picky, I want to understand things inside and out)

Actually the Hoyle Narlikar papers are very difficult to read because they use an unconventional notation, just like Feynman did.  The best thing to do is to go to Hoyle Narlikar's books (*), rather than the papers.  A more straightforward approach is to start with a good General Relativity book and linearize the equations but preserving the time rates that are usually neglected.  If you like to follow this approach, I would suggest you look at Dr. Lance Williams paper in the proceedings of the Estes Workshop (when it comes out).


(*) "Lectures on Cosmology and action at a distance electrodynamics" by Fred Hoyle and Jayant V. Narlikar  world scientific 1996

"Action at a distance in physics and cosmology" Hoyle and Narlikar Freeman and co. (1974).    (recommended !)

EDIT: By the way Dirac used advanced waves in 1938 to derive the standard  theory of radiation reaction of a charged particle, see Jackson E&M.  Einstein was present when Feynman gave his first talk in absorber theory.. See "The Beat of a Different Drum: The Life and Science of Richard Feynman,
by Jagdish Mehra ". So was Bohr. Bohr turned to Einstein and said, this just can't be right Dr Einstein, don't you agree? Einstein is said to have replied... "I just think it would be hard to come up with a gravitational equivalent."  Narlikar and Hoyle (1965) went on to derive a gravitational version of absorber theory where one mass interacts directly with another to produce gravitation.

« Last Edit: 09/26/2016 12:59 am by Rodal »

Offline M.E.T.

  • Senior Member
  • *****
  • Posts: 2313
  • Liked: 2912
  • Likes Given: 508
Re: Woodward's effect
« Reply #765 on: 09/26/2016 08:39 am »
Dr. Rodal

As a long time follower of Woodward's experiments, (and to a lesser extent of the EMdrive which seems to have received the more serious scientific attention in recent years) I find it exciting that you have had the opportunity interact with and contribute to Heidi and Jim's work.

I am curious. Do I sense a warming from your side to the possibilities presented by Prof. Woodward's Mach Effect theories? In your view, could we, in the EM drive and Mach Effect Thruster, potentially be looking at two unrelated mechanisms that might both eventually prove to have value in the area of thrust generation?

I would find this a rather remarkable development, if so.
« Last Edit: 09/26/2016 08:39 am by M.E.T. »

Offline Rodal

  • Senior Member
  • *****
  • Posts: 5911
  • USA
  • Liked: 6124
  • Likes Given: 5564
Re: Woodward's effect
« Reply #766 on: 09/26/2016 01:10 pm »
...could we, in the EM drive and Mach Effect Thruster, potentially be looking at two unrelated mechanisms that might both eventually prove to have value in the area of thrust generation?

I would find this a rather remarkable development, if so.
Perhaps the same mechanism is responsible for both the EM Drive and the piezoelectric/electrostrictive stacks tested by Woodward/Fearn: see

https://forum.nasaspaceflight.com/index.php?topic=40959.msg1588165#msg1588165

https://forum.nasaspaceflight.com/index.php?topic=40959.msg1588214#msg1588214

for Dr. Montillet's paper explaining the EM Drive's acceleration as being due to the Mach/Sciama/Woodward gravitational effect.

...Woodward does not think his Mach drive and the EM drive  operate on the same principle. As of the 1st edition of his book he was skeptical of the EM drive as being anything other than experimental error of some sort. Of course with the cross fertilization that has happened since then (E.G; the SSI conference) he may have changed his opinion on that point. The conference attendees would know if that is still the case or not.
I don't know what was his opinion previously, but the above statement is incorrect as of the Estes Park Breakthrough Workshop, as Prof. Woodward was agreeable to Dr. Montillet's explanation of the EM Drive as being due to the Mach effect.  Actually Prof. Woodward proposed an experiment (that a superconducting EM Drive's performance will be less than expected because of the much reduced skin depth) to test this idea. 
« Last Edit: 09/26/2016 01:16 pm by Rodal »

Offline M.E.T.

  • Senior Member
  • *****
  • Posts: 2313
  • Liked: 2912
  • Likes Given: 508
Re: Woodward's effect
« Reply #767 on: 09/26/2016 01:26 pm »
...could we, in the EM drive and Mach Effect Thruster, potentially be looking at two unrelated mechanisms that might both eventually prove to have value in the area of thrust generation?

I would find this a rather remarkable development, if so.
Perhaps the same mechanism is responsible for both the EM Drive and the piezoelectric/electrostrictive stacks tested by Woodward/Fearn: see

https://forum.nasaspaceflight.com/index.php?topic=40959.msg1588165#msg1588165

https://forum.nasaspaceflight.com/index.php?topic=40959.msg1588214#msg1588214

for Dr. Montillet's paper explaining the EM Drive's acceleration as being due to the Mach/Sciama/Woodward gravitational effect.

...Woodward does not think his Mach drive and the EM drive  operate on the same principle. As of the 1st edition of his book he was skeptical of the EM drive as being anything other than experimental error of some sort. Of course with the cross fertilization that has happened since then (E.G; the SSI conference) he may have changed his opinion on that point. The conference attendees would know if that is still the case or not.
I don't know what was his opinion previously, but the above statement is incorrect as of the Estes Park Breakthrough Workshop, as Prof. Woodward was agreeable to Dr. Montillet's explanation of the EM Drive as being due to the Mach effect.  Actually Prof. Woodward proposed an experiment (that a superconducting EM Drive's performance will be less than expected because of the much reduced skin depth) to test this idea.

Am I interpreting the above correctly? Is the suggestion that the Mach Effect was the underlying cause of the thrust signatures all along, and that the EMdrive theorists' search for alternative explanations might merely lead them back to the Mach Effect in the end?

If so, I believe this is something Prof. Woodward alluded to some years ago. Rather than saying that the EMdrive results were completely flawed, I seem to recall him suggesting originally that if there is any thrust there, it might be due to accidental Mach Effects being generated.

Are we approaching some kind of consensus among those involved in studying both phenomena that this might indeed be the case?

Offline birchoff

  • Full Member
  • **
  • Posts: 273
  • United States
  • Liked: 125
  • Likes Given: 95
Re: Woodward's effect
« Reply #768 on: 09/26/2016 01:42 pm »
...could we, in the EM drive and Mach Effect Thruster, potentially be looking at two unrelated mechanisms that might both eventually prove to have value in the area of thrust generation?

I would find this a rather remarkable development, if so.
Perhaps the same mechanism is responsible for both the EM Drive and the piezoelectric/electrostrictive stacks tested by Woodward/Fearn: see

https://forum.nasaspaceflight.com/index.php?topic=40959.msg1588165#msg1588165

https://forum.nasaspaceflight.com/index.php?topic=40959.msg1588214#msg1588214

for Dr. Montillet's paper explaining the EM Drive's acceleration as being due to the Mach/Sciama/Woodward gravitational effect.

...Woodward does not think his Mach drive and the EM drive  operate on the same principle. As of the 1st edition of his book he was skeptical of the EM drive as being anything other than experimental error of some sort. Of course with the cross fertilization that has happened since then (E.G; the SSI conference) he may have changed his opinion on that point. The conference attendees would know if that is still the case or not.
I don't know what was his opinion previously, but the above statement is incorrect as of the Estes Park Breakthrough Workshop, as Prof. Woodward was agreeable to Dr. Montillet's explanation of the EM Drive as being due to the Mach effect.  Actually Prof. Woodward proposed an experiment (that a superconducting EM Drive's performance will be less than expected because of the much reduced skin depth) to test this idea.

Am I interpreting the above correctly? Is the suggestion that the Mach Effect was the underlying cause of the thrust signatures all along, and that the EMdrive theorists' search for alternative explanations might merely lead them back to the Mach Effect in the end?

If so, I believe this is something Prof. Woodward alluded to some years ago. Rather than saying that the EMdrive results were completely flawed, I seem to recall him suggesting originally that if there is any thrust there, it might be due to accidental Mach Effects being generated.

Are we approaching some kind of consensus among those involved in studying both phenomena that this might indeed be the case?

your recollection is correct. I remember the same comment. However, Woodward was never actually against the EmDrive. He was just against the theory, Dr. White's i believe, that was being used to explain it. I would have to troll through the em drive thread but I believe this was relayed to us by Paul March in one of his ghostly appearances on the forum.

Offline M.E.T.

  • Senior Member
  • *****
  • Posts: 2313
  • Liked: 2912
  • Likes Given: 508
Re: Woodward's effect
« Reply #769 on: 09/26/2016 01:50 pm »
...could we, in the EM drive and Mach Effect Thruster, potentially be looking at two unrelated mechanisms that might both eventually prove to have value in the area of thrust generation?

I would find this a rather remarkable development, if so.
Perhaps the same mechanism is responsible for both the EM Drive and the piezoelectric/electrostrictive stacks tested by Woodward/Fearn: see

https://forum.nasaspaceflight.com/index.php?topic=40959.msg1588165#msg1588165

https://forum.nasaspaceflight.com/index.php?topic=40959.msg1588214#msg1588214

for Dr. Montillet's paper explaining the EM Drive's acceleration as being due to the Mach/Sciama/Woodward gravitational effect.

...Woodward does not think his Mach drive and the EM drive  operate on the same principle. As of the 1st edition of his book he was skeptical of the EM drive as being anything other than experimental error of some sort. Of course with the cross fertilization that has happened since then (E.G; the SSI conference) he may have changed his opinion on that point. The conference attendees would know if that is still the case or not.
I don't know what was his opinion previously, but the above statement is incorrect as of the Estes Park Breakthrough Workshop, as Prof. Woodward was agreeable to Dr. Montillet's explanation of the EM Drive as being due to the Mach effect.  Actually Prof. Woodward proposed an experiment (that a superconducting EM Drive's performance will be less than expected because of the much reduced skin depth) to test this idea.

Am I interpreting the above correctly? Is the suggestion that the Mach Effect was the underlying cause of the thrust signatures all along, and that the EMdrive theorists' search for alternative explanations might merely lead them back to the Mach Effect in the end?

If so, I believe this is something Prof. Woodward alluded to some years ago. Rather than saying that the EMdrive results were completely flawed, I seem to recall him suggesting originally that if there is any thrust there, it might be due to accidental Mach Effects being generated.

Are we approaching some kind of consensus among those involved in studying both phenomena that this might indeed be the case?

your recollection is correct. I remember the same comment. However, Woodward was never actually against the EmDrive. He was just against the theory, Dr. White's i believe, that was being used to explain it. I would have to troll through the em drive thread but I believe this was relayed to us by Paul March in one of his ghostly appearances on the forum.

Yes, the Quantum Vacuum Fluctuation theory in particular was the one he disagreed with I believe. Also, in his audio interview on the Space Show about a year or so ago, which I listened to, he was quite clear in his disagreement with the above.

I just find it exciting that after all of the focus on the EMDrive, and the apparent disregard for Woodward's work during this time (this thread received almost no updates for months on end compared to the multitude of EMDrive threads), things seem to be going full circle, back to his Mach Effect theories once more.

What I'm trying to ascertain is whether this is indeed the case, or whether I am misinterpreting the latest developments.
« Last Edit: 09/26/2016 02:00 pm by M.E.T. »

Offline Star One

  • Senior Member
  • *****
  • Posts: 13997
  • UK
  • Liked: 3974
  • Likes Given: 220
Re: Woodward's effect
« Reply #770 on: 09/26/2016 02:24 pm »
...could we, in the EM drive and Mach Effect Thruster, potentially be looking at two unrelated mechanisms that might both eventually prove to have value in the area of thrust generation?

I would find this a rather remarkable development, if so.
Perhaps the same mechanism is responsible for both the EM Drive and the piezoelectric/electrostrictive stacks tested by Woodward/Fearn: see

https://forum.nasaspaceflight.com/index.php?topic=40959.msg1588165#msg1588165

https://forum.nasaspaceflight.com/index.php?topic=40959.msg1588214#msg1588214

for Dr. Montillet's paper explaining the EM Drive's acceleration as being due to the Mach/Sciama/Woodward gravitational effect.

...Woodward does not think his Mach drive and the EM drive  operate on the same principle. As of the 1st edition of his book he was skeptical of the EM drive as being anything other than experimental error of some sort. Of course with the cross fertilization that has happened since then (E.G; the SSI conference) he may have changed his opinion on that point. The conference attendees would know if that is still the case or not.
I don't know what was his opinion previously, but the above statement is incorrect as of the Estes Park Breakthrough Workshop, as Prof. Woodward was agreeable to Dr. Montillet's explanation of the EM Drive as being due to the Mach effect.  Actually Prof. Woodward proposed an experiment (that a superconducting EM Drive's performance will be less than expected because of the much reduced skin depth) to test this idea.

Am I interpreting the above correctly? Is the suggestion that the Mach Effect was the underlying cause of the thrust signatures all along, and that the EMdrive theorists' search for alternative explanations might merely lead them back to the Mach Effect in the end?

If so, I believe this is something Prof. Woodward alluded to some years ago. Rather than saying that the EMdrive results were completely flawed, I seem to recall him suggesting originally that if there is any thrust there, it might be due to accidental Mach Effects being generated.

Are we approaching some kind of consensus among those involved in studying both phenomena that this might indeed be the case?

your recollection is correct. I remember the same comment. However, Woodward was never actually against the EmDrive. He was just against the theory, Dr. White's i believe, that was being used to explain it. I would have to troll through the em drive thread but I believe this was relayed to us by Paul March in one of his ghostly appearances on the forum.

Yes, the Quantum Vacuum Fluctuation theory in particular was the one he disagreed with I believe. Also, in his audio interview on the Space Show about a year or so ago, which I listened to, he was quite clear in his disagreement with the above.

I just find it exciting that after all of the focus on the EMDrive, and the apparent disregard for Woodward's work during this time (this thread received almost no updates for months on end compared to the multitude of EMDrive threads), things seem to be going full circle, back to his Mach Effect theories once more.

What I'm trying to ascertain is whether this is indeed the case, or whether I am misinterpreting the latest developments.

Well this one has the advantage of not running roughshod over some of the strongest laws in physics so it was always in with a more than equal chance of being the best theoretical explanation.
« Last Edit: 09/26/2016 02:25 pm by Star One »

Offline birchoff

  • Full Member
  • **
  • Posts: 273
  • United States
  • Liked: 125
  • Likes Given: 95
Re: Woodward's effect
« Reply #771 on: 09/26/2016 03:21 pm »
...

Yes, the Quantum Vacuum Fluctuation theory in particular was the one he disagreed with I believe. Also, in his audio interview on the Space Show about a year or so ago, which I listened to, he was quite clear in his disagreement with the above.

I just find it exciting that after all of the focus on the EMDrive, and the apparent disregard for Woodward's work during this time (this thread received almost no updates for months on end compared to the multitude of EMDrive threads), things seem to be going full circle, back to his Mach Effect theories once more.

What I'm trying to ascertain is whether this is indeed the case, or whether I am misinterpreting the latest developments.

While the recent focus has been on the EmDrive. the focus in the EmDrive thread has really shifted to experimentation. Every so often there would be discussions of theory. But the bulk of the correspondence in the thread is really talking about simulations and feedback on builds. I think Dr. Rodal even went as far as spinning up a different thread to expound on his mathematical comments which he would reference in the EmDrive thread.

As for whether or not things are coming full circle. I would say pump the brakes a bit. from everything I have seen Dr. Rodal say. I would say it is more accurate that the list of possible theories that could be used to explain experimental results is getting smaller with Mach Effect being close to the top. In the end, I actually wonder if Mach Effects as described by Woodward doesnt end up being more of a modal of a fraction of reality in the end.

I have in particular always been a fan of his theory given the firmer footing it always seemed to have Theoretically. In addition to it leaving open the possibility of creating traversable worm holes. However, in light of the work Dr. Rodal has recently done on Hoyle Narlikar Theory of Gravitation. Worm holes may end up being a mirage on the horizon. Though if we end up with a legit space drive with the ability to scale to thrust levels seen in current rocket technology and higher. I would gladly make that trade; it just means I will have to invest in the idea of sleeper ships if I ever hope to see humanity leave its mother star.

That said, I really really want to see this linearized GR version of Mach Effects. Mainly becasue I have been operating under the assumption that the only reason Headi adopted HN Theory was because it already had Gravitational absorber theory integrated into it. So if there is a linearized GR version of Mach Effects, one has to wonder if the absorber theory is really neccessary. Would love to hear Dr. Rodals take.

Offline Rodal

  • Senior Member
  • *****
  • Posts: 5911
  • USA
  • Liked: 6124
  • Likes Given: 5564
Re: Woodward's effect
« Reply #772 on: 09/26/2016 03:40 pm »
...That said, I really really want to see this linearized GR version of Mach Effects. Mainly becasue I have been operating under the assumption that the only reason Headi adopted HN Theory was because it already had Gravitational absorber theory integrated into it. So if there is a linearized GR version of Mach Effects, one has to wonder if the absorber theory is really neccessary. Would love to hear Dr. Rodals take.
1.  I think not much can be said about black holes and this theory at the moment.  It is a purely gravitational theory.  No quantum effects.  The theory and experiments is mainly directed towards space propulsion based on gravitational inertial effects and significant work remains ahead just in this very small domain.

2. Dr. Lance Williams has shown that the Mach effect transient terms can be derived from a linearized GR.  That is comforting, certainly.  Whether these solutions are present in Nature has to be resolved by experiments.  The Hoyle-Narlikar derivation from Heidi Fearn is robust from the point of view that she derives it from the fully nonlinear, covariant formulation.  My contribution is to rigorously derive a force from those transient terms.  Action at a distance (advanced waves) is needed to justify this mathematical solution as a physical solution:  under either derivation.
« Last Edit: 09/26/2016 03:44 pm by Rodal »

Offline Tellmeagain

  • Full Member
  • **
  • Posts: 252
  • maryland
  • Liked: 153
  • Likes Given: 34
Re: Woodward's effect
« Reply #773 on: 09/28/2016 02:27 pm »
I have the kindle version. Unfortunately those pages are the book page numbers not the ebook one so I couldnt provide you the information you want.

Thanks for the consideration anyway.

The reason I am asking origintates in the followng quote from woodward's recent monograph

Quote
In the case of a rocket motor, the thing to observe is that there is one invariant
velocity involved: that of the exhaust plume with respect to the motor. All observers,
irrespective of their own motions, agree on both the magnitude and direction of this
velocity. And it is the velocity that yields momentum conservation. An argument based
on an incorrect application of Newton’s second law to METs was advanced as a criticism
of Mach Effects by an Oak Ridge scientist many years ago. It is dealt with on pages 77
and 127 of Making Starships and Stargates: the Science of Interstellar Propulsion and
Absurdly Benign Wormholes.
It will not be discussed further here.

As I understand it, the argument advanced by the Oak Ridge scientist was that F=ma is only correct at the system level for systems where the mass of the system is constant in time (which is clearly not true for METs).  This is because the more general expression for Force is correctly given by (I leave it to the reader to distinguish vectors from scalars):

Fnet=(d/dt)(p)
=d/dt(mv)
=(dm/dt)v + m(dv/dt)
=(dm/dt)v + ma

where:
F=external forces (i.e. forces applied across the system boundary)
p=momentum
m=mass
v=velocity
a=acceleration
d/dt= partial derivative with respect to time

Obviously we can return to the more familiar F=ma expression by just setting mass constant so that dm/dt=0.

To demonstrate that this is the correct expression, consider the case of a rocket expelling exhaust in free space (i.e. no gravity; the only thing in this universe is the rocket and it's exhaust).  Now draw a control volume around the rocket plus a portion of the exhaust stream that has been expelled from the rocket.  Let this control volume be fixed to the rocket so it moves with it (i.e. a lagrangian control volume). 

Now as is always the case, internal forces (f) are always equal and opposite to one another, and so they sum to zero.  Because the "rocket + some exhaust" system is in free space, it experiences no external forces either, so F=0.  Remember that the force applied to the rocket by the exhaust is equal and opposite the force on the exhaust applied by the rocket.  Since we have included this portion of the exhaust in our control volume, these forces are themselves internal, and sum to zero (see attached figure). 

Now if we applied F=ma we would get that 0=ma so a=0.  Apparently rockets don't work. 

But if we apply the correct force derivation, F=d/dt(p) then we get:

Fnet=(dm/dt)v + ma
0=(dm/dt)v + ma
a=((-dm/dt)v)/m

which is the correct equation of motion for a rocket.  Essentially, when we choose a control volume where a portion of the exhaust is in the control volume and that portion has been fully accelerated (such that any particle of exhaust has a=0), then the acceleration of a rocket is NOT in fact from a "force" at all, but from the expulsion of mass across the system boundary that envelops the control volume (at least in the lagrangian interpretation).

This preamble has to do with the MET because mass fluctuations only lead to an apparent force within a closed system IF you take the INCORRECT F=ma approach as opposed to the more general F=d/dt(p) approach.  If you have a closed system (i.e. nothing transfers across the system boundary) and you apply cyclic mass variations within the system, you would see that:

Fnet=(dm/dt)v + ma
0=(dm/dt)v + ma                       (Fnet=0 because the system is closed)
0=0 + ma                                   (since there is no flow of mass across the system boundary)***
a=0

And so (following this derivation) cyclic mass variation won't actually yield any propulsive benefits.

Basically I want to see how Woodward addresses this issue, and whether it is done so in such a way as to make METs viable.  I want to see why Woodward finds this argument "incorrect" as he says.  Welcome other posters to point out any issues with this derivation as well of course. 



***If there is energy flux across the system boundary in the form of gravinertial waves, then those would account for the dm/dt term and presumably gravinertial waves travel at the speed of light, so v=c.  However, if that were the case, then the MET would be unable to achieve thrust greater than a perfectly collimated photon rocket, which can be derived as follows:

E=mc2
m=E/c2
(d/dt)m=(d/dt)E/c2
dm/dt=P/c2

E=energy
P=power
c=speed of light

Sub this expression for dm/dt into the rocket equation (realizing that we have now replaced the exhaust flow with photons/gluons/W,Z bosons, gravitons, gravienterial waves, magic energy waves - literally anything that has energy but no rest mass) and you get:

0=(P/c2)c + ma
ma=P/c

So we are back to the thrust of a perfectly collimated photon rocket.

I wrote an analysis to Professor Woodward's monograph linked in your post. I posted it in the EmDrive thread,
http://forum.nasaspaceflight.com/index.php?topic=40959.msg1589319#msg1589319
(link to pdf available in that post)
There I included a section talking about when the equation dp/dt=Mdv/dt vdM/dt holds and when it does not hold. I think it is relevant to your discussion.


Offline birchoff

  • Full Member
  • **
  • Posts: 273
  • United States
  • Liked: 125
  • Likes Given: 95
Re: Woodward's effect
« Reply #774 on: 09/29/2016 01:35 pm »
I have the kindle version. Unfortunately those pages are the book page numbers not the ebook one so I couldnt provide you the information you want.

Thanks for the consideration anyway.

The reason I am asking origintates in the followng quote from woodward's recent monograph

Quote
In the case of a rocket motor, the thing to observe is that there is one invariant
velocity involved: that of the exhaust plume with respect to the motor. All observers,
irrespective of their own motions, agree on both the magnitude and direction of this
velocity. And it is the velocity that yields momentum conservation. An argument based
on an incorrect application of Newton’s second law to METs was advanced as a criticism
of Mach Effects by an Oak Ridge scientist many years ago. It is dealt with on pages 77
and 127 of Making Starships and Stargates: the Science of Interstellar Propulsion and
Absurdly Benign Wormholes.
It will not be discussed further here.

As I understand it, the argument advanced by the Oak Ridge scientist was that F=ma is only correct at the system level for systems where the mass of the system is constant in time (which is clearly not true for METs).  This is because the more general expression for Force is correctly given by (I leave it to the reader to distinguish vectors from scalars):

Fnet=(d/dt)(p)
=d/dt(mv)
=(dm/dt)v + m(dv/dt)
=(dm/dt)v + ma

where:
F=external forces (i.e. forces applied across the system boundary)
p=momentum
m=mass
v=velocity
a=acceleration
d/dt= partial derivative with respect to time

Obviously we can return to the more familiar F=ma expression by just setting mass constant so that dm/dt=0.

To demonstrate that this is the correct expression, consider the case of a rocket expelling exhaust in free space (i.e. no gravity; the only thing in this universe is the rocket and it's exhaust).  Now draw a control volume around the rocket plus a portion of the exhaust stream that has been expelled from the rocket.  Let this control volume be fixed to the rocket so it moves with it (i.e. a lagrangian control volume). 

Now as is always the case, internal forces (f) are always equal and opposite to one another, and so they sum to zero.  Because the "rocket + some exhaust" system is in free space, it experiences no external forces either, so F=0.  Remember that the force applied to the rocket by the exhaust is equal and opposite the force on the exhaust applied by the rocket.  Since we have included this portion of the exhaust in our control volume, these forces are themselves internal, and sum to zero (see attached figure). 

Now if we applied F=ma we would get that 0=ma so a=0.  Apparently rockets don't work. 

But if we apply the correct force derivation, F=d/dt(p) then we get:

Fnet=(dm/dt)v + ma
0=(dm/dt)v + ma
a=((-dm/dt)v)/m

which is the correct equation of motion for a rocket.  Essentially, when we choose a control volume where a portion of the exhaust is in the control volume and that portion has been fully accelerated (such that any particle of exhaust has a=0), then the acceleration of a rocket is NOT in fact from a "force" at all, but from the expulsion of mass across the system boundary that envelops the control volume (at least in the lagrangian interpretation).

This preamble has to do with the MET because mass fluctuations only lead to an apparent force within a closed system IF you take the INCORRECT F=ma approach as opposed to the more general F=d/dt(p) approach.  If you have a closed system (i.e. nothing transfers across the system boundary) and you apply cyclic mass variations within the system, you would see that:

Fnet=(dm/dt)v + ma
0=(dm/dt)v + ma                       (Fnet=0 because the system is closed)
0=0 + ma                                   (since there is no flow of mass across the system boundary)***
a=0

And so (following this derivation) cyclic mass variation won't actually yield any propulsive benefits.

Basically I want to see how Woodward addresses this issue, and whether it is done so in such a way as to make METs viable.  I want to see why Woodward finds this argument "incorrect" as he says.  Welcome other posters to point out any issues with this derivation as well of course. 



***If there is energy flux across the system boundary in the form of gravinertial waves, then those would account for the dm/dt term and presumably gravinertial waves travel at the speed of light, so v=c.  However, if that were the case, then the MET would be unable to achieve thrust greater than a perfectly collimated photon rocket, which can be derived as follows:

E=mc2
m=E/c2
(d/dt)m=(d/dt)E/c2
dm/dt=P/c2

E=energy
P=power
c=speed of light

Sub this expression for dm/dt into the rocket equation (realizing that we have now replaced the exhaust flow with photons/gluons/W,Z bosons, gravitons, gravienterial waves, magic energy waves - literally anything that has energy but no rest mass) and you get:

0=(P/c2)c + ma
ma=P/c

So we are back to the thrust of a perfectly collimated photon rocket.

I wrote an analysis to Professor Woodward's monograph linked in your post. I posted it in the EmDrive thread,
http://forum.nasaspaceflight.com/index.php?topic=40959.msg1589319#msg1589319
(link to pdf available in that post)
There I included a section talking about when the equation dp/dt=Mdv/dt vdM/dt holds and when it does not hold. I think it is relevant to your discussion.


I had considered this discussion to be concluded. with the result being that WallOfWolfStreet having a problem that Woodward has not shown why fluctuating the mass allows him to zero out the vdM/dt term of the general equation. However, in light of new comments made about this issue. I realized that this discussion was predetermined to ending the way it did. The reason for this is neither I or any of the people who understand Woodward's theory enough believe you can draw the box to do your CoE/CoM calculation around only the MET. If you do you will end up with a violation. Woodward's Theory plainly calls for an energy exchange with the rest of the mass in the universe. I didn't have a good way of describing this before but Dr. Rodal's description of this being a type of Gravity assist I think is very apt. The only difference is you wouldn't need to sling your way around a gravitating body to gain that momentum and instead of leveraging one Gravitating mass you are leveraging all of it, chiefly the stuff very far away.

Now the impasse WallOfWolfstreet and I reached about the zeroing out of the vdM/dt term probably still exists. But I think until we have a better understanding of what happens to the mass of the FMC (Fluctuating Mass Component) part of the MET during the oscillations. I don't think a firm answer of the kind that Wall would like can be put forward. For now, the only thing that can be offered is experimental results that so far agree with the theoretical predictions of not only Woodward's original Thrust equation. But also with additional refinements of that equation. Where the refinements continue to reduce the delta between what is predicted and what is observed.

The first set of Refinements came, as I understand it, from Headi Fearn rederiving Mach Effects from Hoyle and Narlikar Theory of Gravitation while answering the critique that Stephen Hawking had raised about HN Theory. As of this month I learned from Dr. Rodal
...

2. Dr. Lance Williams has shown that the Mach effect transient terms can be derived from a linearized GR.  That is comforting, certainly.  Whether these solutions are present in Nature has to be resolved by experiments.  The Hoyle-Narlikar derivation from Heidi Fearn is robust from the point of view that she derives it from the fully nonlinear, covariant formulation.  My contribution is to rigorously derive a force from those transient terms.  Action at a distance (advanced waves) is needed to justify this mathematical solution as a physical solution:  under either derivation.

I believe Dr. Rodal has also left comments in either this thread of the EmDrive thread recently referring to successful replications being done by other experimenters outside Woodward's lab. Now, while the inner space geek in me is pretty convinced this works. My more rational side is aware of the fact that there is still work to be done. I don't think anyone is going to really accept any Mach Effect theory be it derived from HN Theory or Linearized GR until there is a high level of agreement between Experiment and predictions; they do have some of that now but the confidence bars need to further be increased.

Offline Star One

  • Senior Member
  • *****
  • Posts: 13997
  • UK
  • Liked: 3974
  • Likes Given: 220
Re: Woodward's effect
« Reply #775 on: 09/29/2016 04:30 pm »
I have the kindle version. Unfortunately those pages are the book page numbers not the ebook one so I couldnt provide you the information you want.

Thanks for the consideration anyway.

The reason I am asking origintates in the followng quote from woodward's recent monograph

Quote
In the case of a rocket motor, the thing to observe is that there is one invariant
velocity involved: that of the exhaust plume with respect to the motor. All observers,
irrespective of their own motions, agree on both the magnitude and direction of this
velocity. And it is the velocity that yields momentum conservation. An argument based
on an incorrect application of Newton’s second law to METs was advanced as a criticism
of Mach Effects by an Oak Ridge scientist many years ago. It is dealt with on pages 77
and 127 of Making Starships and Stargates: the Science of Interstellar Propulsion and
Absurdly Benign Wormholes.
It will not be discussed further here.

As I understand it, the argument advanced by the Oak Ridge scientist was that F=ma is only correct at the system level for systems where the mass of the system is constant in time (which is clearly not true for METs).  This is because the more general expression for Force is correctly given by (I leave it to the reader to distinguish vectors from scalars):

Fnet=(d/dt)(p)
=d/dt(mv)
=(dm/dt)v + m(dv/dt)
=(dm/dt)v + ma

where:
F=external forces (i.e. forces applied across the system boundary)
p=momentum
m=mass
v=velocity
a=acceleration
d/dt= partial derivative with respect to time

Obviously we can return to the more familiar F=ma expression by just setting mass constant so that dm/dt=0.

To demonstrate that this is the correct expression, consider the case of a rocket expelling exhaust in free space (i.e. no gravity; the only thing in this universe is the rocket and it's exhaust).  Now draw a control volume around the rocket plus a portion of the exhaust stream that has been expelled from the rocket.  Let this control volume be fixed to the rocket so it moves with it (i.e. a lagrangian control volume). 

Now as is always the case, internal forces (f) are always equal and opposite to one another, and so they sum to zero.  Because the "rocket + some exhaust" system is in free space, it experiences no external forces either, so F=0.  Remember that the force applied to the rocket by the exhaust is equal and opposite the force on the exhaust applied by the rocket.  Since we have included this portion of the exhaust in our control volume, these forces are themselves internal, and sum to zero (see attached figure). 

Now if we applied F=ma we would get that 0=ma so a=0.  Apparently rockets don't work. 

But if we apply the correct force derivation, F=d/dt(p) then we get:

Fnet=(dm/dt)v + ma
0=(dm/dt)v + ma
a=((-dm/dt)v)/m

which is the correct equation of motion for a rocket.  Essentially, when we choose a control volume where a portion of the exhaust is in the control volume and that portion has been fully accelerated (such that any particle of exhaust has a=0), then the acceleration of a rocket is NOT in fact from a "force" at all, but from the expulsion of mass across the system boundary that envelops the control volume (at least in the lagrangian interpretation).

This preamble has to do with the MET because mass fluctuations only lead to an apparent force within a closed system IF you take the INCORRECT F=ma approach as opposed to the more general F=d/dt(p) approach.  If you have a closed system (i.e. nothing transfers across the system boundary) and you apply cyclic mass variations within the system, you would see that:

Fnet=(dm/dt)v + ma
0=(dm/dt)v + ma                       (Fnet=0 because the system is closed)
0=0 + ma                                   (since there is no flow of mass across the system boundary)***
a=0

And so (following this derivation) cyclic mass variation won't actually yield any propulsive benefits.

Basically I want to see how Woodward addresses this issue, and whether it is done so in such a way as to make METs viable.  I want to see why Woodward finds this argument "incorrect" as he says.  Welcome other posters to point out any issues with this derivation as well of course. 



***If there is energy flux across the system boundary in the form of gravinertial waves, then those would account for the dm/dt term and presumably gravinertial waves travel at the speed of light, so v=c.  However, if that were the case, then the MET would be unable to achieve thrust greater than a perfectly collimated photon rocket, which can be derived as follows:

E=mc2
m=E/c2
(d/dt)m=(d/dt)E/c2
dm/dt=P/c2

E=energy
P=power
c=speed of light

Sub this expression for dm/dt into the rocket equation (realizing that we have now replaced the exhaust flow with photons/gluons/W,Z bosons, gravitons, gravienterial waves, magic energy waves - literally anything that has energy but no rest mass) and you get:

0=(P/c2)c + ma
ma=P/c

So we are back to the thrust of a perfectly collimated photon rocket.

I wrote an analysis to Professor Woodward's monograph linked in your post. I posted it in the EmDrive thread,
http://forum.nasaspaceflight.com/index.php?topic=40959.msg1589319#msg1589319
(link to pdf available in that post)
There I included a section talking about when the equation dp/dt=Mdv/dt vdM/dt holds and when it does not hold. I think it is relevant to your discussion.


I had considered this discussion to be concluded. with the result being that WallOfWolfStreet having a problem that Woodward has not shown why fluctuating the mass allows him to zero out the vdM/dt term of the general equation. However, in light of new comments made about this issue. I realized that this discussion was predetermined to ending the way it did. The reason for this is neither I or any of the people who understand Woodward's theory enough believe you can draw the box to do your CoE/CoM calculation around only the MET. If you do you will end up with a violation. Woodward's Theory plainly calls for an energy exchange with the rest of the mass in the universe. I didn't have a good way of describing this before but Dr. Rodal's description of this being a type of Gravity assist I think is very apt. The only difference is you wouldn't need to sling your way around a gravitating body to gain that momentum and instead of leveraging one Gravitating mass you are leveraging all of it, chiefly the stuff very far away.

Now the impasse WallOfWolfstreet and I reached about the zeroing out of the vdM/dt term probably still exists. But I think until we have a better understanding of what happens to the mass of the FMC (Fluctuating Mass Component) part of the MET during the oscillations. I don't think a firm answer of the kind that Wall would like can be put forward. For now, the only thing that can be offered is experimental results that so far agree with the theoretical predictions of not only Woodward's original Thrust equation. But also with additional refinements of that equation. Where the refinements continue to reduce the delta between what is predicted and what is observed.

The first set of Refinements came, as I understand it, from Headi Fearn rederiving Mach Effects from Hoyle and Narlikar Theory of Gravitation while answering the critique that Stephen Hawking had raised about HN Theory. As of this month I learned from Dr. Rodal
...

2. Dr. Lance Williams has shown that the Mach effect transient terms can be derived from a linearized GR.  That is comforting, certainly.  Whether these solutions are present in Nature has to be resolved by experiments.  The Hoyle-Narlikar derivation from Heidi Fearn is robust from the point of view that she derives it from the fully nonlinear, covariant formulation.  My contribution is to rigorously derive a force from those transient terms.  Action at a distance (advanced waves) is needed to justify this mathematical solution as a physical solution:  under either derivation.

I believe Dr. Rodal has also left comments in either this thread of the EmDrive thread recently referring to successful replications being done by other experimenters outside Woodward's lab. Now, while the inner space geek in me is pretty convinced this works. My more rational side is aware of the fact that there is still work to be done. I don't think anyone is going to really accept any Mach Effect theory be it derived from HN Theory or Linearized GR until there is a high level of agreement between Experiment and predictions; they do have some of that now but the confidence bars need to further be increased.

As to you last point I can confirm that I also remember Dr. Rodal mentioning recently there has been further successful replications which came out of the SSI conference.

Offline Star One

  • Senior Member
  • *****
  • Posts: 13997
  • UK
  • Liked: 3974
  • Likes Given: 220
Re: Woodward's effect
« Reply #776 on: 09/30/2016 06:39 am »
CUBESAT MISSION CLARIFICATION

Quote
There has been a lot of erroneous information in media articles regarding Cannae’s upcoming launch of a cubesat mission into LEO. To clarify our previous post and press release: Cannae is not using an EmDrive thruster in our upcoming launch. Cannae is using it’s own proprietary thruster technology which requires no on-board propellant to generate thrust. In addition, this project is being done as a private venture. Cannae is only working with our private commercial partners on the upcoming mission.

http://cannae.com/cubesat-mission-clarification/

Offline Rodal

  • Senior Member
  • *****
  • Posts: 5911
  • USA
  • Liked: 6124
  • Likes Given: 5564
Re: Woodward's effect
« Reply #777 on: 12/03/2016 06:46 pm »
I am periodically updating drafts of my article on Mach Effect Propulsion, a chapter in the proceedings of the 2016 Advanced Propulsion Workshop at Estes Park Colorado, to be published in the near future.

If you like, you can download my report from here:

https://www.researchgate.net/project/Mach-effect-propulsion

Best regards

Offline M.E.T.

  • Senior Member
  • *****
  • Posts: 2313
  • Liked: 2912
  • Likes Given: 508
Re: Woodward's effect
« Reply #778 on: 12/03/2016 08:19 pm »
I am periodically updating drafts of my article on Mach Effect Propulsion, a chapter in the proceedings of the 2016 Advanced Propulsion Workshop at Estes Park Colorado, to be published in the near future.

If you like, you can download my report from here:

https://www.researchgate.net/project/Mach-effect-propulsion

Best regards

Thank you, Dr. Rodal.

I read through it - skipping past the maths which are way beyond me.

From your expert point of view, would you say that there is something going on here? Obviously you state all the further areas that require investigation. But given the knowledge that exists now, is there a way to build a scaled up version of Woodward's lab device, which could prove or disprove whether there is an effect once and for all, by generating thrusts of useful magnitudes?
« Last Edit: 12/03/2016 08:23 pm by M.E.T. »

Offline aceshigh

  • Full Member
  • ****
  • Posts: 792
  • Liked: 269
  • Likes Given: 22
Re: Woodward's effect
« Reply #779 on: 12/14/2016 01:37 am »
I am periodically updating drafts of my article on Mach Effect Propulsion, a chapter in the proceedings of the 2016 Advanced Propulsion Workshop at Estes Park Colorado, to be published in the near future.

If you like, you can download my report from here:

https://www.researchgate.net/project/Mach-effect-propulsion

Best regards

Thank you, Dr. Rodal.

I read through it - skipping past the maths which are way beyond me.

From your expert point of view, would you say that there is something going on here? Obviously you state all the further areas that require investigation. But given the knowledge that exists now, is there a way to build a scaled up version of Woodward's lab device, which could prove or disprove whether there is an effect once and for all, by generating thrusts of useful magnitudes?

quoting MultiVac, MicroVac, Galactic AC and UniversalAC: "THERE IS INSUFFICIENT DATA FOR A MEANINGFUL ANSWER."

:)

ps: The Last Question. Isaac Asimov

Tags:
 

Advertisement NovaTech
Advertisement Northrop Grumman
Advertisement
Advertisement Margaritaville Beach Resort South Padre Island
Advertisement Brady Kenniston
Advertisement NextSpaceflight
Advertisement Nathan Barker Photography
1